Quantum Mechanics – Perturbation Theory

A particle of mass M is in an infinitely deep square well potential   where

      for    ,         and
    for    .

A very small perturbing potential '  is superimposed on   such that

      for            , and

                         for       .

If      are the energy eigenfunctions for a particle in the infinitely deep square well potential, with      being the ground state, which of the following statements is correct about the eigenfunction     of a particle in the perturbed potential ?

A.      

B.     style="vertical-align: -8pt;"   with        for all odd values of n.

C.       with        for all even values of n.

D.        with       for all values of n.

E.    None of the above
(GR8677 #96)
Solution:

The eigenfunctions of the unperturbed infinite deep well form a complete set (or complete basis).

This means that any function can be represented by the old unperturbed infinite deep well.

Thus, the solution to the perturbed eigenfunction should look like .



Also, since the perturbed potential is also symmetrical with respect to the origin (as the original unperturbed potential was, too), one knows that all the odd terms should go to 0.

Answer: B


No comments :